Administración     

Olimpiadas de Matemáticas
Página de preparación y problemas

OME Local
OME Andaluza
OME Nacional
OIM
IMO
EGMO
USAMO
ASU
OMCC
Retos UJA
Selector
La base de datos contiene 2434 problemas y 940 soluciones.

V Olimpiada Iberoamericana de Matemáticas — 1990

Sesión 1 —  26 de septiembre de 1990

Problema 568
Sea $f$ una función definida en el conjunto $\mathbb{N}_0$ de los números enteros mayores o iguales que cero y que satisface las siguientes condiciones:
  • Si $n=2^j-1$ para algún $j\in\mathbb{N}_0$, entonces $f(n)=0$.
  • Si $n\neq 2^j-1$ para todo $j\in\mathbb{N}_0$, entonces $f(n+1)=f(n)-1$.
Demostrar que, para todo $n\in\mathbb{N}_0$, existe $k\in\mathbb{N}_0$ tal que $f(n)+n=2^k-1$. Calcular $f(2^{1990})$.
pistasolución 1info
Pista. Observa que los valores de la función entre $2^j$ y $2^{j+1}-1$ decrecen de unidad en unidad desde $f(2^j)=2^j-1$ hasta $f(2^{j+1}-1)=0$.
Solución. Para cada $k\in\mathbb N$, el intervalo $[2^k,2^{k+1}-2]$ no contiene números de la forma $2^j-1$, luego el valor de la función $f$ en cada número de ese intervalo es una unidad mayor que en el número siguiente. Como $f(2^{k+1}-1)=0$, se sigue que los valores de $f$ decrecen de unidad en unidad desde $f(2^k)=2^k-1$ hasta $f(2^{k+1}-1)=0$. En otras palabras, tenemos que $f(2^k+m)=2^k-m-1$ para todo entero $0\leq m\leq 2^k-1$, lo que determina unívocamente a la función $f$ ya que todo entero positivo se expresa de forma única como $2^k+m$ con $k,m\in\mathbb{N}_0$ y $0\leq m\leq 2^k-1$.

Esto responde a la primera pregunta ya que, si $n=2^k+m$ con $0\leq m\leq 2^k-1$, entonces \[f(n)+n=2^k-m-1+2^k+m=2^{k+1}-1.\] Además $f(0)+0=f(2^0-1)=0=2^0-1$, luego la propiedad también se cumple para $n=0$. Para responder a la segunda pregunta, expresamos $2^{1990}=2^k+m$ con $k=1990$ y $m=0$, luego \[f(2^{1990})=2^{1990}-0-1=2^{1990}-1.\]

Si crees que el enunciado contiene un error o imprecisión o bien crees que la información sobre la procedencia del problema es incorrecta, puedes notificarlo usando los siguientes botones:
Informar de error en enunciado Informar de procedencia del problema
Problema 657
En un triángulo $ABC$, sean $I$ el centro de la circunferencia inscrita y $D$, $E$ y $F$ sus puntos de tangencia con los lados $BC$, $AC$ y $AB$, respectivamente. Sea $P$ el otro punto de intersección de la recta $AD$ con la circunferencia inscrita. Si $M$ es el punto medio de $EF$ , demostrar que los cuatro puntos $P$, $I$, $M$ y $D$ pertenecen a una misma circunferencia o están alineados.
pistasolución 1info
Pista. Utiliza la potencia de $A$ respecto de la circunferencia inscrita y respecto de una hipotética circunferencia que pasa por esos cuatro puntos.
Solución. Como la circunferencia inscrita es tangente al lado $CA$ en $F$, el radio $IF$ es perpendicular a $CA$, luego el triángulo $AFI$ (sombreado en azul) es rectángulo. El segmento $FM$ es una de sus alturas, con lo que $FMI$ es semejante a $AFI$ y esto nos da la relación $\frac{AF}{AM}=\frac{AI}{AF}$, luego $AF^2=AI\cdot AM$. Ahora bien, por la potencia respecto de la circunferencia inscrita, se tiene también que $AF^2=AP\cdot AD$ y, por tanto, $AP\cdot AD=AI\cdot AM$. Usando el recíproco de la propiedad de la potencia (del punto $A$), esto nos dice que $P,I,M,D$ están sobre la misma circunferencia (siempre que no estén alineados).imagen

Nota. Los puntos están alineados cuando la bisectriz del ángulo $A$ pasa por el punto $D$, lo cual es equivalente a que el triángulo $ABC$ sea isósceles (con ángulo desigual en $A$).

Si crees que el enunciado contiene un error o imprecisión o bien crees que la información sobre la procedencia del problema es incorrecta, puedes notificarlo usando los siguientes botones:
Informar de error en enunciado Informar de procedencia del problema
Problema 477
Sea $f(x)=(x+b)^2-c$ un polinomio con $b$ y $c$ números enteros.
  1. Si $p$ es un número primo que divide a $c$ y $p^2$ no divide a $c$, demostrar que $p^2$ no divide a $f(n)$ para ningún entero $n\in\mathbb{Z}$.
  2. Sea $q$ un número primo distinto de $2$ que no divide a $c$. Si $q$ divide a $f(n)$ para algún entero $n\in\mathbb{Z}$, demostrar que para cada entero positivo $r$ existe un entero $n'$ tal que $q^r$ divide a $f(n')$.
pistasolución 1info
Pista. Razona el apartado (a) por reducción al absurdo y el apartado (b) por inducción sobre $r$.
Solución. El primer apartado es sencillo razonando por reducción al absurdo. Si existiera un valor de $n\in\mathbb{Z}$ tal que $p^2|f(n)=(n+b)^2-c$, entonces $p|(n+b)^2$ ya que $p|c$ por hipótesis. Como se trata de un cuadrado, necesariamente $p^2|(n+b)^2$, luego también se sigue que $p^2|c=(n+b)^2-f(n)$. Esto contradice la hipótesis de que $p^2$ no divide a $c$.

En cuanto al apartado (b), vamos a proceder por inducción sobre $r$. Para $r=1$, no hay nada que probar ya que tenemos la hipótesis de que $q|f(n)$ para algún $n\in\mathbb Z$. Dado $r\geq 1$, supongamos que $q^r|(n+b)^2-c$ para algún $n\in\mathbb Z$ y probemos que existe $n'\in\mathbb Z$ tal que $q^{r+1}|(n'+b)^2-c$. Vamos a elegir $n'=n+aq^r$ para cierto $a\in\mathbb{Z}$ que vamos a determinar a continuación. Esto nos da \[f(n')=(n+aq^r+b)^2-c=(n+b)^2-c+2aq^r(n+b)+a^2q^{2r}=(d+2a(n+b))q^r+a^2q^{2r},\] donde hemos escrito $(n+b)^2-c=dq^r$ para cierto $d\in\mathbb{Z}$ usando la hipótesis de inducción. Por tanto, habremos terminado si probamos que la ecuación en congruencias $2a(n+b)\equiv -d\ (\text{mod }q)$ tiene solución (siendo $a$ la incógnita). Esto se deduce de que $2(n+b)$ tiene inverso módulo $q$ ya que $\mathrm{mcd}(2(n+b),q)=1$. Esto último se deduce a su vez de que $q$ divide a $f(n)$ pero no a $c$ (luego no $n+b$ no puede ser múltiplo de $q$) y de que $q\neq 2$ por hipótesis.

Si crees que el enunciado contiene un error o imprecisión o bien crees que la información sobre la procedencia del problema es incorrecta, puedes notificarlo usando los siguientes botones:
Informar de error en enunciado Informar de procedencia del problema

Sesión 2 —  27 de septiembre de 1990

Problema 658
Sea $C_1$ una circunferencia, $AB$ uno de sus diámetros, $t$ su recta tangente en $B$ y $M$ un punto de $C_1$ distinto de $A$ y de $B$. Se construye una circunferencia $C_2$ tangente a $C_1$ en $M$ y a la recta $t$.
  1. Determinar el punto $P$ de tangencia de $t$ y $C_2$ y hallar el lugar geométrico de los centros de las circunferencias $C_2$ al variar $M$.
  2. Demostrar que existe una circunferencia ortogonal a todas las circunferencias $C_2$.
pistasolución 1info
Pista. La solución al apartado (a) es una cierta cónica. Para el apartado (b), una inversión simplifica considerablemente el problema.
Solución. Sean $O_1$ y $O_2$ los centros de $C_1$ y $C_2$, respectivamente, y $r_1$ y $r_2$ sus radios. Si consideramos la recta $s$ paralela a $t$ a distancia $R_1$ y que no contiene a $O$, se tiene que la distancia de $O_2$ a $O_1$ es igual a $r_2+r_1$, que es la distancia de $O_2$ a $s$. Por lo tanto, el lugar geométrico de $O_2$ es una parábola que tiene por vértice $B$. Está claro además que $O_2O_1=r_2+r_1$ es la hipotenusa de un triángulo rectángulo de catetos $BP$ y $|r_1-r_2|$, de donde podemos despejar \[BP=\sqrt{(r_1+r_2)^2-(r_1-r_2)^2}=2\sqrt{r_1r_2},\] luego podemos determinar $P$ como el punto de $t$ que está a distancia $2\sqrt{r_1r_2}$ de $B$ y en la misma semirrecta en que se proyecta $O_2$.

Para responder al apartado (b), hacemos una inversión respecto de la circunferencia centrada en $B$ y de radio $2r_1$, que deja $t$ invariante y lleva $C_1$ en la recta $C_1'$ paralela a $t$ que pasa por $A$. Como las inversiones mantienen las rectas/circunferencias, los ángulos y las tangencias, la circunferencia $C_2$ se transforma en otra circunferencia $C_2'$ tangente a las rectas paralelas $C_1'$ y $t$. Entonces, la recta $C'$ paralela a $t$ que pasa por $O$ es ortogonal a todas las circunferencias $C_2'$. Tras invertir de nuevo (deshacemos la inversión), $C'$ se transforma en la circunferencia $C$ tangente a $t$ en $B$ y de radio $2r_1$, y es ortogonal a todas las circunferencias $C_2$.

Si crees que el enunciado contiene un error o imprecisión o bien crees que la información sobre la procedencia del problema es incorrecta, puedes notificarlo usando los siguientes botones:
Informar de error en enunciado Informar de procedencia del problema
Problema 659
Sean $A$ y $B$ los vértices opuestos de un tablero cuadriculado de $n\times n$ casillas ($n\geq 1$), a cada una de las cuales se añade su diagonal en la dirección $AB$, formándose así $2n^2$ triángulos iguales. Se mueve una ficha recorriendo un camino que va desde $A$ hasta $B$ formado por segmentos del tablero y se coloca, cada vez que se recorre un segmento, una semilla en cada uno de los triángulos que admiten ese segmento como lado. El camino se recorre de tal forma que no se pasa por ningún segmento más de una vez, y se observa, después del recorrido, que hay exactamente dos semillas en cada uno de los $2n^2$ triángulos del tablero. ¿Para qué valores de $n$ es posible esta situación?
pistasolución 1info
Pista. El problema se reduce a eliminar segmentos de forma que se elimine exactamente un lado de cada triángulo y el grafo resultante se pueda recorrer desde $A$ hasta $B$ pasando una única vez por cada segmento. Razona lo que debe ocurrir en una esquina distinta de $A$ y $B$.
Solución. Si eliminamos los segmentos que no se recorren nos queda un grafo por el que puede viajarse de $A$ a $B$ recorriendo cada segmento exactamente una vez, lo cual puede hacerse si el grafo es euleriano, es decir, si en $A$ y $B$ llegan un número impar de aristas y a cualquier otro vértice llegan un número par. Por otro lado, para que queden dos semillas en cada triángulo se debe pasar exactamente por dos lados del triángulo. Por lo tanto, el problema equivale a quitar una cierta cantidad de segmentos de forma que:
  • Se quite exactamente un lado de cada uno de los $2n^2$ triángulos.
  • A todos los vértices, excepto $A$ y $B$, llegue un número par de segmentos.

El caso $n=1$ claramente es imposible pero el caso $n=2$ sí se puede, como indica la figura superior (hemos indicado en línea discontinua los segmentos que se eliminan). Para el caso de $n\geq 3$, comenzaremos razonando en una esquina que no es $A$ ni $B$, que supondremos que es la inferior izquierda y etiquetaremos las aristas como indica en la figura central. Las aristas $c_1$ y $a_1$ no se pueden eliminar ya que a la esquina deben llegar un número par de aristas y no se pueden eliminar dos lados de ese triángulo. Por lo tanto, es $d_1$ la que debe eliminarse. También debe eliminarse $a_2$ porque, en caso contrario, $d_1$, $a_1$ y $a_2$ llegarían a un vértice con un número impar de aristas. Esto nos lleva a que debe conservarse $d_2$. Análogamente, debe eliminarse $e_1$ y, por tanto, $f_1$ también debe conservarse. Ahora bien, si dejáramos $c_3$, entonces $b_2$ se eliminaría, luego $e_2$ también se eliminaría (por la paridad del vértice) y el triángulo de lados $e_2,b_2,f_2$ tendría dos aristas eliminadas; esto nos dice que $c_3$ también se elimina, luego $b_2$ debe conservarse, $f_2$ eliminarse y $e_3$ conservarse. Más aún, $a_3$ y $d_3$ deben conservarse porque $c_3$ se ha eliminado. El proceso puede continuarse con el mismo razonamiento para demostrar que todos los $c_n$ con $n\geq 3$ y todos los $f_n$ con $n\geq 2$ se eliminan, quedando un patrón de cuadrados y paralelogramos como se muestra en la figura inferior. No obstante, cuando llegamos al vértice $B$ esto produce una contradicción ya que $a_n$, $b_n$ y $d_n$ deben permanecer. Como a $B$ deben llegar un número impar de aristas, también $c_{n+1}$ debe conservarse, pero esto nos da un triángulo en el que no se ha eliminado ninguna arista.

Deducimos así que el único caso posible es $n=2$.

imagen
Si crees que el enunciado contiene un error o imprecisión o bien crees que la información sobre la procedencia del problema es incorrecta, puedes notificarlo usando los siguientes botones:
Informar de error en enunciado Informar de procedencia del problema
Problema 660
Sea $f(x)$ un polinomio de grado $3$ con coeficientes racionales. Si la gráfica de $f(x)$ es tangente al eje de abscisas en algún punto, demostrar que $f(x)$ tiene sus tres raíces racionales.
pistasolución 1solución 2info
Pista. Expresa las ecuaciones de Cardano-Vieta e intenta expresar las raíces en términos de los coeficientes.
Solución. Pongamos que el polinomio es $f(x)=x^3+ax^2+bx+c$, ya que podemos suponer sin pérdida de generalidad que el coeficiente líder es $1$. Que sea tangente al eje de abscisas implica que debe tener una raíz doble $r$ y pongamos que $s$ es la tercera raíz (podría ser $r=s$ y tener una raíz triple). Entonces, igualando coeficientes en \[x^3+ax^2+bx+c=(x-r)^2(x-s)=x^3-(2r+s)x^2+(r^2+2rs)x-r^2s,\] deducimos que $2r+s=-a$, $r^2+2rs=b$ y $r^2s=-c$ son números racionales (estas son las ecuaciones de Cardano-Vièta). Buscando combinaciones de estos números que nos den expresiones simétricas y homogéneas de $r$ y $s$, encontramos sin mucho problema \[a^2-3b=r^2-2rs+s^2,\qquad 9c-ab=2r(r^2-2rs+s^2).\] Distinguimos dos casos:
  • Si $a^2-3b\neq 0$, entonces obtenemos directamente que \[r=\frac{9c-ab}{2(a^2-3b)}\in\mathbb{Q},\qquad s=-a-2r=-\frac{a^3+9c-4ab}{a^2-3b}\in\mathbb{Q}.\]
  • En caso contrario, tenemos que $0=a^2-3b=(2r+s)^2-3(r^2+2rs)=(r-s)^2$, luego se trata de un polinomio con una raíz triple, que también es racional ya que podemos escribir en este caso $r=s=\frac{-a}{3}\in\mathbb{Q}$.
Solución. Consideremos la derivada $f'(x)$, que es un polinomio de grado $2$ con coeficientes racionales. Que la gráfica sea tangente al eje de abscisas en algún punto nos dice que debe existir $\alpha\in\mathbb{R}$ tal que $f(\alpha)=f'(\alpha)=0$ ($\alpha$ es una raíz doble o triple de $f(x)$). La división de $f(x)$ entre $f'(x)$ nos asegura que existen polinomios $q(x)$ y $r(x)$ con coeficientes racionales tales que $f(x)=q(x)f'(x)+r(x)$, luego evaluando en $x=\alpha$ obtenemos que $r(\alpha)=0$. Distingamos dos casos:
  • Si $r(x)=ax+b$ es un polinomio de grado $1$ ($a\neq 0$), como tiene coeficientes racionales, podemos despejar $\alpha=\frac{-b}{a}\in\mathbb{Q}$. Ahora basta dividir $f(x)$ entre $(x-\alpha)^2$ para obtener otro polinomio $x-\beta$ que también debe tener coeficientes racionales, y claramente $\beta\in\mathbb{Q}$ es la tercera raíz de $f(x)$.
  • Si $r(x)$ es de grado menor que $1$, entonces debe ser necesariamente $r(x)=0$ (ya que $r(\alpha)=0$). Entonces, $f(x)$ es divisible entre $f'(x)$. Como cualquier raíz común a $f(x)$ y $f'(x)$ tiene multiplicidad en $f(x)$ una unidad más que en $f'(x)$, se deduce que $f'(x)$ no puede tener dos raíces distintas, luego en este caso $\alpha$ es una raíz triple de $f(x)$. Si escribimos entonces $f(x)=a(x-\alpha)^3$ para cierto número racional $a\neq 0$, podemos despejar $\alpha$ como el coeficiente de $x^2$ de $f(x)$ dividido por $-3a$, luego la raíz triple $\alpha$ también es racional en este caso.
Si crees que el enunciado contiene un error o imprecisión o bien crees que la información sobre la procedencia del problema es incorrecta, puedes notificarlo usando los siguientes botones:
Informar de error en enunciado Informar de procedencia del problema
José Miguel Manzano © 2010-2025. Esta página ha sido creada mediante software libre